The answer to this question surprised me -- Please Help..

This topic has expert replies
Junior | Next Rank: 30 Posts
Posts: 21
Joined: Fri Mar 09, 2012 12:44 pm
Economist: On average, the emergency treatment for an elderly person for injuries resulting from a fall costs $11,000. A new therapeutic program can significantly reduce an elderly person's chances of falling. Though obviously desirable for many reasons, this treatment program will cost $12,500 and thus cannot be justified.

Which of the following, if true, most seriously undermines the conclusion of the argument?

A. Among elderly people who had followed the program for only a few months, the number of serious falls reported was higher than it was for people who had followed the program for its recommended minimum length of one year.
B. Falls resulting in serious injuries are less common among elderly people living in nursing homes than they are among elderly people who live alone at home.
C. A frequent result of injuries sustained in falls is long-term pain, medication for which is not counted among the average per-person costs of emergency treatment for elderly people's injuries from such falls.
D. The new therapeutic program focuses on therapies other than medication, since overmedication can cause disorientation and hence increase the likelihood that an elderly person will have a serious fall.
E. A significant portion of the cost of the new therapeutic program is represented by regular visits by health care professionals, the costs of which tend to increase more rapidly than do those of other elements of the program.

Was confused b/w D and E. Went for D but found the right answer to be C

Please help how should I think to arrive at the right answer for such question.

Thanks for the help..

I do see one thread for this question (https://www.beatthegmat.com/q-from-gmatprep-t45529.html) but no explanation is provided. Explanations by experts would really help.

Legendary Member
Posts: 581
Joined: Sun Apr 03, 2011 7:53 am
Thanked: 52 times
Followed by:5 members

by killer1387 » Tue Apr 24, 2012 7:23 am
Economist: On average, the emergency treatment for an elderly person for injuries resulting from a fall costs $11,000. A new therapeutic program can significantly reduce an elderly person's chances of falling. Though obviously desirable for many reasons, this treatment program will cost $12,500 and thus cannot be justified.

Which of the following, if true, most seriously undermines the conclusion of the argument?

A. Among elderly people who had followed the program for only a few months, the number of serious falls reported was higher than it was for people who had followed the program for its recommended minimum length of one year.
--> Irrelevant

B. Falls resulting in serious injuries are less common among elderly people living in nursing homes than they are among elderly people who live alone at home.
--> irrelevant

C. A frequent result of injuries sustained in falls is long-term pain, medication for which is not counted among the average per-person costs of emergency treatment for elderly people's injuries from such falls.
--> correct as it attacks the premise saying the figure based on which conclusion is being derived is actually more than that.

D. The new therapeutic program focuses on therapies other than medication, since overmedication can cause disorientation and hence increase the likelihood that an elderly person will have a serious fall.
--> this option talks of overmedication and doesnt speaks of therapies. we actually dont know what are the result of therapies and also we cant assume overmedication frequently

E. A significant portion of the cost of the new therapeutic program is represented by regular visits by health care professionals, the costs of which tend to increase more rapidly than do those of other elements of the program.
--> going extreme this option would support the conclusion/ anyway we can immediately reject this option as it doesnt matter what is the composition and variation of 12500. we are focussing on the conclusion stating new program is futile.

Hope this helps..!!

Junior | Next Rank: 30 Posts
Posts: 21
Joined: Fri Mar 09, 2012 12:44 pm

by gauraku » Tue Apr 24, 2012 7:35 am
Thanks buddy!!

Closely going through your explanation and the premise, the answer made sense to me. If the cost quoted as 11,000$ is actually more than that, we can say that the cost of new therapy definitely makes sense.

Also as u mentioned, D says about overmedication and not therapy makes me feel that I should go through the premise and the options more closely.

User avatar
Master | Next Rank: 500 Posts
Posts: 287
Joined: Fri Mar 23, 2012 12:33 am
Location: Pune,India
Thanked: 60 times
Followed by:6 members

by GMAT Kolaveri » Tue Apr 24, 2012 8:08 am
Economist: On average, the emergency treatment for an elderly person for injuries resulting from a fall costs $11,000. A new therapeutic program can significantly reduce an elderly person's chances of falling. Though obviously desirable for many reasons, this treatment program will cost $12,500 and thus cannot be justified.

Argument Analysis
Emergency treatment (option 1) - Costs $11,000
New program(option 2) - will cost $12,500
Conclusion: Option 2 is costlier than option 1 and thus cannot be justified.

Prethinking:
The author draws the conclusion from the inequality option2 > option1.
To weaken this we have to show that option2 is cheaper than option1.
Different ways to weaken the conclusion:
1. Person undergoing option 1 treatment requires lot of medications, nurse care.these costs are not included in calculation of emergency treatment(option 1).
2. Person undergoing option 1 treatment cannot resume work for X months where person undergoing option 2 treatment can resume work immediately. hence the cost of treat can be justified.[in this case elderly persons cannot be expected to resume work :) ]
3. If its for a certain disease, we can weaken by saying that option 2 is a permanent cure and option 1 is temporary cure. the higher cost for option 2 can be justified in this case.

In going thro' the AOs, only C weakens the conclusion: option2 > option1

Also pay close attention to words such as average and other quantifier words
Regards and Thanks,
Vinoth@GMAT Kolaveri
https://www.facebook.com/GmatKolaveri
https://gmatkolaveri.tumblr.com/

Click the thank you button if you like my reply :)

User avatar
Legendary Member
Posts: 1239
Joined: Tue Apr 26, 2011 6:25 am
Thanked: 233 times
Followed by:26 members
GMAT Score:680

by sam2304 » Tue Apr 24, 2012 8:27 am
Both of the above users have explained it very clearly. The key lies in understanding the meaning of the argument and what you are supposed to do. 'and thus cannot be justified.' - this is important in deciding what you are supposed to do.

Program X costs 11k, program y prevents the cause and costs 12.5k but is costly. You are asked to weaken the conclusion that program Y is of no use. Once you come to this point you can easily choose C.
Getting defeated is just a temporary notion, giving it up is what makes it permanent.
https://gmatandbeyond.blogspot.in/

Newbie | Next Rank: 10 Posts
Posts: 2
Joined: Wed Nov 07, 2012 10:05 pm

by [email protected] » Sat Jan 12, 2013 7:32 am
Hi,

I do not agree with the explanation provided for C to be wrong, C does not say that the cost for medication for longterm pain is significant, it can be less amount as well may be $200 in which case the cost of therapeutic treatment will still be higher. It is said that we should not assume when reviewing answer choices, why are we assuming that that cost of medication is significant enough to make the other option attractive?

D seems to be better choice, since it says that the emergency treatment cost is not the one time cost, I mean this treatment increases the possibility of fall and that means another 11,500 cost but on the other hand therapeutic treatment does not opens up such possibilities.

Kindly comment where I am wrong especially about the logic for C?

Regards,

GMAT/MBA Expert

User avatar
GMAT Instructor
Posts: 3380
Joined: Mon Mar 03, 2008 1:20 am
Thanked: 2256 times
Followed by:1535 members
GMAT Score:800

by lunarpower » Sat Jan 12, 2013 10:20 pm
when you consider these problems, the single most important thing is to stay focused on the ISSUE that underlies the argument.

in this case, it's pretty straightforward to define the issue: the argument is about dollars and cents, so, at the end of the day, financial costs are the only real issue here.

basically, here's the situation:
we know for sure that 11000 < 12500; this is not something that can be questioned. so, if you want to damage this argument, you have to find something that will swing this inequality the other way. by definition, that's going to have to be one of two things:
1/
some other, hidden cost of the emergency treatment that's not inherent in the original $11000 price tag (and that would therefore raise the real price of emergency treatment)
... or ...
2/
some hidden financial savings that's not inherent in the original $12500 price tag (and that would therefore lower the real cost of the treatment program)

choice (c) is the definite winner here, since it satisfies #1 here exactly. (in fact, the problem even spells out that the extra cost is "not included in the cost of emergency treatment".)

choice (d) doesn't help, because, if anything, it just provides an extra reason to believe something that's already stated in the passage.
in particular, the passage already tells us that the new therapeutic program can significantly reduce an elderly person's chances of falling.
choice (d) provides a nice reason WHY the program might reduce those chances -- but it's already stated as a fact that those chances are reduced! thus, the specific reasons why don't help; those are already factored into the statement made in the argument.
Ron has been teaching various standardized tests for 20 years.

--

Pueden hacerle preguntas a Ron en castellano
Potete chiedere domande a Ron in italiano
On peut poser des questions à Ron en français
Voit esittää kysymyksiä Ron:lle myös suomeksi

--

Quand on se sent bien dans un vêtement, tout peut arriver. Un bon vêtement, c'est un passeport pour le bonheur.

Yves Saint-Laurent

--

Learn more about ron

GMAT/MBA Expert

User avatar
GMAT Instructor
Posts: 3380
Joined: Mon Mar 03, 2008 1:20 am
Thanked: 2256 times
Followed by:1535 members
GMAT Score:800

by lunarpower » Sat Jan 12, 2013 10:28 pm
[email protected] wrote:I do not agree with the explanation provided for C to be wrong, C does not say that the cost for medication for longterm pain is significant, it can be less amount as well may be $200 in which case the cost of therapeutic treatment will still be higher. It is said that we should not assume when reviewing answer choices, why are we assuming that that cost of medication is significant enough to make the other option attractive?
don't forget -- these are "strengthen/weaken" problems. they are NOT "confirm/destroy" problems!

you will very, very rarely be given evidence that spells things out so exactly as to establish things beyond any doubt.
that's not the point of these problems. the point is to understand what other factors can affect an issue, at least significantly enough to make a difference.
in this passage, the issue is dollars and cents, and choice (c) is the only option with a direct bearing (and in the right direction) on that issue.
D seems to be better choice, since it says that the emergency treatment cost is not the one time cost, I mean this treatment increases the possibility of fall and that means another 11,500 cost but on the other hand therapeutic treatment does not opens up such possibilities.
nope.

choice (d) connects medication with additional costs. however, the passage gives no reason at all to connect "emergency treatment" and "medication".

if you are making a connection between emergency treatment and medication, then you are accidentally importing option (c) into your reasoning. that's a mistake. don't forget what's in the passage and what isn't!
Ron has been teaching various standardized tests for 20 years.

--

Pueden hacerle preguntas a Ron en castellano
Potete chiedere domande a Ron in italiano
On peut poser des questions à Ron en français
Voit esittää kysymyksiä Ron:lle myös suomeksi

--

Quand on se sent bien dans un vêtement, tout peut arriver. Un bon vêtement, c'est un passeport pour le bonheur.

Yves Saint-Laurent

--

Learn more about ron

Newbie | Next Rank: 10 Posts
Posts: 2
Joined: Wed Nov 07, 2012 10:05 pm

by [email protected] » Sun Jan 13, 2013 9:31 am
Thank you for the explanation. I got it now. But what I do not understand is there are always blue areas in critical reasoning questions. I have sort of mathematical mind in which x has definite values so not good for such problems,

I have solved more than 150 CR but my accuracy is stuck at 60% and timing is 4 min per CR, can you please guide me how to improve it? I have e-Gmat course and clear about all the concepts for CR but when it comes to solving question, I narrow down to two choices and chose the wrong one.

Can you give me a strategy for improving CR?

Thank you so much,

Regards,

Junior | Next Rank: 30 Posts
Posts: 16
Joined: Fri Mar 29, 2013 11:30 am
Thanked: 1 times

by ggmat007 » Wed May 01, 2013 3:08 am
How about option E. As Ron mentioned - 2/
some hidden financial savings that's not inherent in the original $12500 price tag (and that would therefore lower the real cost of the treatment program) - Doesn't E mentions the hidden costs.

C is right. Can't question that but please help me understand what is wrong with E.

Newbie | Next Rank: 10 Posts
Posts: 3
Joined: Mon Apr 15, 2013 5:18 am
Thanked: 1 times
GMAT Score:740

by ssbisht » Wed May 01, 2013 9:46 pm
ggmat007 wrote:How about option E. As Ron mentioned - 2/
some hidden financial savings that's not inherent in the original $12500 price tag (and that would therefore lower the real cost of the treatment program) - Doesn't E mentions the hidden costs.

C is right. Can't question that but please help me understand what is wrong with E.
Remember we shouldn't assume anything in CR.
In choice E it is not mentioned if the cost of visits by health care professional can be reduced significantly and hence we can't conclude anything from it.
Thanks
If you find my post helpful, please take a moment to click on the "Thank" icon.

GMAT Instructor
Posts: 2630
Joined: Wed Sep 12, 2012 3:32 pm
Location: East Bay all the way
Thanked: 625 times
Followed by:119 members
GMAT Score:780

by Matt@VeritasPrep » Wed May 01, 2013 9:54 pm
ggmat007 wrote:How about option E. As Ron mentioned - 2/
some hidden financial savings that's not inherent in the original $12500 price tag (and that would therefore lower the real cost of the treatment program) - Doesn't E mentions the hidden costs.

C is right. Can't question that but please help me understand what is wrong with E.
Good question! (E) doesn't really strengthen or weaken the argument, it just tells you more about something you already know: the therapeutic program is expensive, and now you know one reason why. (Nothing in (E) implies that you can reduce these costs or that these visits are expendable, so you don't want to assume that you can/they are.)

On strengthen and weaken questions this sort of answer is always tempting but seldom good; further background on WHY a premise is true doesn't often strengthen or weaken an argument, at least not on the GMAT (which mostly cares WHAT impact the truth of a premise has on an argument).

GMAT/MBA Expert

User avatar
GMAT Instructor
Posts: 3380
Joined: Mon Mar 03, 2008 1:20 am
Thanked: 2256 times
Followed by:1535 members
GMAT Score:800

by lunarpower » Thu May 02, 2013 6:04 pm
Matt@VeritasPrep wrote:...
further background on WHY a premise is true doesn't often strengthen or weaken an argument, at least not on the GMAT (which mostly cares WHAT impact the truth of a premise has on an argument).
In general, well said.

It should be noted, though, that there are arguments whose entire point is to discuss the reasons why particular facts are (or aren't) true. (A GMATPrep problem about "scientists over forty" comes to mind; you could probably google it.) On those problems, the reasons behind the facts are, of course, important.
Otherwise, though, it's pretty much as you've said.
Ron has been teaching various standardized tests for 20 years.

--

Pueden hacerle preguntas a Ron en castellano
Potete chiedere domande a Ron in italiano
On peut poser des questions à Ron en français
Voit esittää kysymyksiä Ron:lle myös suomeksi

--

Quand on se sent bien dans un vêtement, tout peut arriver. Un bon vêtement, c'est un passeport pour le bonheur.

Yves Saint-Laurent

--

Learn more about ron